LSAT and Law School Admissions Forum

Get expert LSAT preparation and law school admissions advice from PowerScore Test Preparation.

 Administrator
PowerScore Staff
  • PowerScore Staff
  • Posts: 8917
  • Joined: Feb 02, 2011
|
#43092
Please post your questions below!
 Tw07
  • Posts: 7
  • Joined: Feb 15, 2020
|
#73913
In the first and second diagrammed set ups it is possible for R and T to be last, but in the third it is not. Is that why E is correct? I was thinking because it IS possible at all, then it could be true. Could you explain?
 Adam Tyson
PowerScore Staff
  • PowerScore Staff
  • Posts: 5153
  • Joined: Apr 14, 2011
|
#73914
This question is about more than just what variable goes last, Tw07. It's about the combination of these two being 4th and 5th, and one of these pairs in those positions will force us to break a rule. R can certainly go last, but if it is that would mean that T is before R, which would also mean that T must be before G (the contrapositive of the second rule). If T is 4th and R is 5th, then T is before R, but G is before T, and that breaks the second rule! That's why E is the correct answer - it cannot be true, while the other four are all possible.
User avatar
 SGD2021
  • Posts: 72
  • Joined: Nov 01, 2021
|
#94598
Hello, what would be the quickest way to do a question like this during the exam? Also, for answer choice C, would this be an accurate representation of how that answer choice could work out: G/H H/G/S G/S R T?

Thank you!
 Adam Tyson
PowerScore Staff
  • PowerScore Staff
  • Posts: 5153
  • Joined: Apr 14, 2011
|
#94605
The quickest and best way to answer this question would be to do a main diagram that shows all the possible relationships, SGD2021. I wouldn't want to do a bunch of local diagrams testing each answer choice here, as that is almost never an efficient approach.

I happen to think this is a great game for drawing templates, and there would be three of them. In one template, S and T would both be before H, and then H before R. Since that also forces T to be before R, T would also have to be before G. This templates would help us to eliminate just one answer choice, answer B, because we can see that H and R could be 4th and 5th.

Another template would put G before T, triggering the second rule. In that case, with H before R, and R, S, and G all before T, T would have to be last, and the 4th variable could be anything other than H. That eliminates answers A and C.

Finally, a template in which S and T are both after H but T is before G, so that the second rule is not triggered. So H would be first, R and S could be anywhere after that, and T could be anywhere after H and before G. This eliminates answer D, because if G is last then any of R, S, or T could be 4th.

Try drawing those templates out and see how helpful they are once you get to the questions! The key to making this a template game, in my opinion, is the binary nature of the last rule. There are only two options - either H is before S and T, or else it is after S and T. From there, lots of other things fall into place, and all the solutions can be accounted for within those three diagrams.

Get the most out of your LSAT Prep Plus subscription.

Analyze and track your performance with our Testing and Analytics Package.